Which one of the following rangers CANNOT be assigned to area 3?

aseikhon11 on September 4, 2020

Why K & not J?

area 1: J area 2: LP area 3: MKO doesn't break any of the rules? but if you put J in area 3 then it breaks rules? area 1: K area 2: LP area 3: MJO (J&K are split because O is not 2) (L should be with M or K but isn't, so J cant be in area 3) please explain

Reply
Create a free account to read and take part in forum discussions.

Already have an account? log in

Victoria on September 21, 2020

Hi @aseikhon11,

Happy to help!

Answer choice (E) is incorrect because there is at least one possible scenario where J can be assigned to Area 3.

There is no rule saying that L cannot be assigned to Area 1, so let's make that switch in your diagram above.

Area 1: LK
Area 2: P
Area 3: MJO

Notice that this meets all of our conditions:

1) M is assigned to Area 3
2) Neither O (3) nor P (2) is assigned to Area 1.
3) L is assigned to the same area as K and not M
4) O is not assigned to Area 2 and J is assigned to a different Area than K.

Your first scenario breaks Rule 3 because L is not assigned to the same area as K or M.

If we put K in Area 3, then both K and M are assigned to Area 3. This violates Rule 3 either way. If we place L in Area 3, then they are assigned to the same Area as both K and M. If we place L in Area 1 or 2, they are not assigned with either K or M.

Hope this helps! Please let us know if you have any further questions.